Incorrect Logical reasoning questions

¡Supera tus tareas y exámenes ahora con Quizwiz!

A 1955 analysis of paint samples from an Italian painting found evidence of cobalt, suggesting the use of cobalt blue, a pigment not used in Europe before 1804. The painting was thus deemed to have been produced sometime after 1804. A 2009 analysis also found cobalt, but that analysis suggested that the painting might have been produced before 1804. Which one of the following, if true, most helps to resolve the apparent discrepancy in the information above? A The 2009 analysis revealed that cobalt was located only in the topmost paint layer, which was possibly applied to conceal damage to original paint layers. B The 2009 analysis used sophisticated scientific equipment that can detect much smaller amounts of cobalt than could the equipment used for the 1955 analysis. C The 2009 analysis took more samples from the painting than the 1955 analysis did, though those samples were smaller. D Many experts, based on the style and the subject matter of the painting, have dated the painting to the 1700s. E New information that came to light in the 1990s suggested that cobalt blue was used only rarely in Italy in the years immediately following 1804.

Correct Answer: A The 2009 analysis revealed that cobalt was located only in the topmost paint layer, which was possibly applied to conceal damage to original paint layers. Question Type: PARADOX*** Rule: The correct answer to a Paradox question will provide a satisfying explanation to this puzzling set of circumstances. No longer shall we be puzzled! We shall see the truth and be blessed by its shining light (or some such uplifting nonsense).

The consensus among astronomers, based upon observations of the surfaces of pulsars, is that pulsars are spinning balls of neutrons compressed into a sphere some 10 kilometers in diameter with a mass roughly equal to that of our sun. However, their observed properties are also consistent with some pulsars actually being filled with quarks, the building blocks of neutrons. Because the core of a quark-filled pulsar, unlike a neutron-filled one, would have an overall positive charge, it would attract a layer of negatively charged particles that could support a crust of neutrons. The statement that the core of a quark-filled pulsar would have an overall positive charge plays which one of the following roles in the argument above? A It helps explain how pulsars could have neutrons on their surface even if they were not entirely made up of neutrons. B It forms part of a challenge to the claim that some pulsars may be made up of quarks. C It helps explain why some pulsars would not be readily recognized as such by astronomers. D It presents a new finding that challenges the consensus view of the structure of pulsars. E It points out a problem with the view that pulsars have a mass roughly equal to that of our sun.

Correct Answer: Question Type: Reasoning***

A six-month public health campaign sought to limit the spread of influenza by encouraging people to take precautions such as washing their hands frequently and avoiding public places when they experience influenza symptoms. Since the incidence of influenza was much lower during those months than experts had predicted, the public evidently heeded the campaign. Which one of the following, if true, most strengthens the argument? A The incidence of food-borne illnesses, which can be effectively controlled by frequent hand washing, was markedly lower than usual during the six-month period. B During the six-month period, the incidence of the common cold, which has many of the same symptoms as influenza, was about the same as usual. C There were fewer large public gatherings than usual during the six-month period. D Independently of the public health campaign, the news media spread the message that one's risk of contracting influenza can be lessened by frequent hand washing. E In a survey completed before the campaign began, many people admitted that they should do more to limit the spread of influenza.

Correct Answer: A The incidence of food-borne illnesses, which can be effectively controlled by frequent hand washing, was markedly lower than usual during the six-month period. Question Type: Strengthen Rule:

Analyst: Any new natural-gas-powered electrical generation station needs to be located close to a natural-gas pipeline, a large body of water for cooling, and transmission lines. It also must be situated in a region where residents will not oppose construction. Our country has an extensive system of transmission lines, but our natural-gas pipelines run in the vicinity of only three of our large bodies of water, and residents would oppose any significant construction projects near these bodies of water. The analyst's statements, if true, most strongly support which one of the following statements about the analyst's country?A It helps explain how pulsars could have neutrons on their surface even if they were not entirely made up of neutrons. B It forms part of a challenge to the claim that some pulsars may be made up of quarks. C It helps explain why some pulsars would not be readily recognized as such by astronomers. D It presents a new finding that challenges the consensus view of the structure of pulsars. E It points out a problem with the view that pulsars have a mass roughly equal to that of our sun.

Correct Answer: E Many residents who would oppose the construction of a new natural-gas-powered electrical station in their region would not oppose the construction of new transmission lines there. Question type: Supported***** Rule: Which one is best supported by the given facts? Ideally, we need one that has been proven. There should be a way to weasel out of the four wrong answers. The right answer is the one that will make you shrug and go "yeah, this one definitely has to be true." The answer that is best supported by the given facts is C. It's too easy to weasel out from under all the rest.

Astronomer: This country's space agency is currently building a new space telescope that is, unfortunately, way over budget. Some people argue that the project should be canceled. But that would be a mistake. If we cancel it now, all the money that has already been spent—which is more than the additional cost required to complete the project—would be wasted. Which one of the following principles, if valid, would most help to justify the astronomer's argument? A A government agency should not cancel a partially completed project unless the amount of money already spent on the project is small relative to the agency's overall budget. B If more than half of the total cost of a project has already been spent, then the project should be completed. C If it becomes clear that the total cost of a project will be more than twice the amount originally budgeted, then the project should be canceled. D One should not commit additional funding to a project just because one has spent considerable money on it in the past. E In determining which scientific projects to fund, governments should give priority to the projects that are most likely to lead to important new discoveries.

Correct answer B. If more than half of the total cost of a project has already been spent, then the project should be completed. Question type- STRENGTHEN ** Rule - Makes the conclusion a fact. ( correct answer will never introduce new concepts or things that are too strong) B. will make the argument win, and it is a sufficient assumption ( prove) side note- A is a trap answer. Main issue - We need the answer choice that helps to prove the astronomers argument.

Environmentalist: Efforts to attain an overall reduction in carbon use by convincing people to focus on their personal use of fossil fuels cannot achieve that goal. Even if most people changed their behavior, changes in personal use of fossil fuels cannot produce the needed reductions in carbon use. Only government policies can produce change on the required scale. The environmentalist's argument requires assuming which one of the following? A Convincing most people to focus on their personal use of fossil fuels would not lead to their successfully pressuring the government into implementing policies that reduce carbon use. B The calculations needed to determine how best to minimize one's personal use of fossil fuels are too difficult for individuals to carry out on their own. C Efforts to convince people to focus on reducing their personal use of fossil fuels have been made only by those who are not currently involved in framing government policy. D It is easier to convince the government to change its policies on carbon use than to convince people to reduce their personal use of fossil fuels. E People who are concerned about environmental issues are more likely to support political candidates who support environmental issues.

Correct answer: A. Convincing most people to focus on their personal use of fossil fuel would not lead to their successfully pressuring the government into implementing policies that reduce carbon use Question type: Necessary assumption***

A six-month public health campaign sought to limit the spread of influenza by encouraging people to take precautions such as washing their hands frequently and avoiding public places when they experience influenza symptoms. Since the incidence of influenza was much lower during those months than experts had predicted, the public evidently heeded the campaign. Which one of the following, if true, most strengthens the argument? A The incidence of food-borne illnesses, which can be effectively controlled by frequent hand washing, was markedly lower than usual during the six-month period. B During the six-month period, the incidence of the common cold, which has many of the same symptoms as influenza, was about the same as usual. C There were fewer large public gatherings than usual during the six-month period. D Independently of the public health campaign, the news media spread the message that one's risk of contracting influenza can be lessened by frequent hand washing. E In a survey completed before the campaign began, many people admitted that they should do more to limit the spread of influenza.

Correct answer: A The incidence of food-borne illnesses, which can be effectively controlled by frequent hand washing, was markedly lower than usual during the six-month period. Question type: Strengthen Rule: The argument concludes that the public health campaign was why influenza incidence was much lower than experts predicted. But do we know for sure that there's not some other explanation? What if a lot more people got the flu shot? What if the experts were way off in their prediction? There are lots of other possible explanations. Anything that rules out one or more alternative explanations would strengthen the argument. Additional evidence supporting the public health campaign as the explanation would also strengthen the argument.

The caffeine in coffee stimulates the production of irritating acid in the stomach. But darker roasts of coffee, produced by roasting the coffee beans longer, contain more N-methylpyridinium (NMP) than lighter roasts, and NMP tends to suppress production of acid in the stomach. Therefore if you drink caffeinated coffee, darker roasts will irritate your stomach less than lighter roasts. The answer to which one of the following questions most helps in evaluating the argument? A Does extending the roasting time of coffee beans increase the amount of caffeine present in the brewed coffee? B Does a reduction in acid production in the stomach have an adverse effect on stomach function? C Would coffee drinkers who drink caffeinated coffee increase their coffee consumption if the coffee they drank contained less caffeine? D Do some coffee drinkers who switch from lighter to darker roasts of coffee increase their daily coffee consumption? E Do lighter roasts of coffee have any important health benefits that darker roasts of coffee lack?

Correct answer: A. Does extending the roasting time of coffee beans increase the amount of caffeine present in the brewed coffee? Question Type: Evaluate ***** Rule: Caffeine stimulates irritating acid production. But dark roast coffee has more NMP than light roast, which "tends to suppress" acid production in the stomach. So, if you drink caffeinated coffee, dark roast will irritate your stomach less than light roast. Hmm. But what if the long roasting process also ups the amount of caffeine, so that the neutralizing NMP just makes dark roast equivalent, stomach-pain wise, to light roast? We need to ask a question helpful in evaluating the conclusion that dark roast is easier on the stomach than light. I would ask, "What does roasting do to caffeine content?" - If the answer to A is no, the argument seems chill; if the answer is yes, the argument is wrecked. Because this is crucial to the argument's viability, this is the most helpful question to ask and, therefore, our answer.

A study at a company found that most meetings showed diminishing returns after 30 minutes, and little could be expected after 60 minutes. Moreover, the most productive meetings were those for which a clear time frame was established. For a meeting at the company to achieve maximum productivity, then, it needs to have a clear time frame and be no more than 30 minutes long. Which one of the following most accurately expresses the conclusion drawn in the argument? A In general, a meeting at the company that is no more than 30 minutes long and has a clear time frame will achieve maximum productivity. B Most meetings at the company show diminishing returns after 30 minutes, according to a study. C A meeting at the company will be maximally productive only if it has a clear time frame and lasts no more than 30 minutes. D According to a study, meetings at the company were the most productive when they had clear time frames. E A study of meetings at the company says that little productivity should be expected after the 60-minute mark.

Correct answer: C. A meeting at the company will be maximally productive only if it has a clear time frame and lasts no more than 30 minutes Question Type: Conclusion*** Rule: The last sentence is the conclusion. We just need to sort through the answer choices and find one that accurately captures this sentence without adding in anything new or different or extra. A. This answer confuses sufficient for necessary. The argument said that keeping meetings short and having a clear timeframe were necessary for maximum productivity. (If you don't do them, you can't be maximally productive.) But the argument did not say that if you do those two things you're guaranteed maximum productivity, which is what A incorrectly says. This is wrong is because there could be any number of things that prevent maximum productivity at any particular meeting. (What if there's an earthquake? What if the boss has real bad gas and everyone can't stop laughing?) If you picked A, you fell for the LSAT's most common flaw. Make sure you understand this flaw and you'll save yourself buckets of points.

A survey published in a leading medical journal in the early 1970s found that the more frequently people engaged in aerobic exercise, the lower their risk of lung disease tended to be. Since other surveys have confirmed these results, it must be the case that aerobic exercise has a significant beneficial effect on people's health. The reasoning above is questionable because the argument The reasoning above is questionable because the argument A ignores anecdotal evidence and bases its conclusion entirely on scientific research B considers only surveys published in one particular medical journal C concludes merely from the fact that two things are correlated that one causes the other D presumes, without providing justification, that anyone who does not have lung disease is in good health E fails to consider that even infrequent aerobic exercise may have some beneficial effect on people's health

Correct answer: C. Concludes merely from the fact that two things are correlated that one causes the other. Question type: Flaw Rule: Demon user Gee L says, I thought there's a disconnect between lung diseases and good health, therefore D. Any comments? Lung disease is clearly bad for your health. Nobody can disagree with that. The argument doesn't say that lung disease is the be-all, end-all of health—it says that avoiding lung disease is a significant health benefit. That's just common sense, not a disconnect. Anyway, this argument exhibits the LSAT's second most common flaw: Inferring causation from correlation. Tune into this flaw now and save yourself a lot of misery and missed questions in the future. They test this all the damn time. When you see correlation to causation, that's the answer. Period. I incorrectly picked D. this is why it is incorrect: The argument does not assume that as long as you don't have lung disease, you're fine. This would be the answer if it was like "as long as you don't get lung disease, you'll live forever." That's not what it did. We can't pick this because it misdescribes the argument.

There are only two plausible views about where the aesthetic value of a painting lies: either in its purely formal qualities or in what the painting means. But there exists no compelling general account of how a painting could derive its value from its purely formal characteristics. Therefore, the aesthetic value of a painting lies in what it means. The pattern of questionable reasoning in the argument above is most similar to that in which one of the following? A This cardiac patient could be treated with surgery or angioplasty, among other methods. But his weak condition would make recovery from surgery a very long process. Therefore, the doctors ought to perform angioplasty. B Should the company be outbid on the new project, it will either have to lay off workers or find new business. But it does not expect to find new business in the foreseeable future. Therefore, it must be expecting to win the bid on the new project. C History is driven primarily by economic forces or primarily by political forces. But no historian has shown convincingly that history is driven mainly by economic forces. Therefore, it is driven primarily by political forces. D Some analysts are forecasting that if the economy expands, the inflation rate will rise or the unemployment rate will fall. But the unemployment rate promises to remain stable. Therefore, the inflation rate will not change either. E If the party does not change its policies, it will lose heavily in the next election. But if it changes its policies, some people who support those policies will be upset and will sit out the next election. Therefore, it is impossible for the party to win in the next election.

Correct answer: C. History is driven by economic forces by political forces. But no historian has shown convincingly that history is driven mainly by economic forces. Therefore, it is driven primarily by political forces. Question Type: Parallel***

Film historians often find it difficult to determine typical audience members' responses to particular films, especially those from the early twentieth century. Box office figures help little, for they indicate only a film's financial success or failure; they do not show what audiences found funny, or frightening, or moving. These historians also find that newspaper and magazine reviews fail to provide much insight. Which one of the following is most strongly supported by the statements above? A Newspaper and magazine reviews of films are usually written in advance of a film's general release. B Typical audience members' responses to films from the latter part of the twentieth century are easy to determine. C The box office success of a film does not depend on its viewers finding it funny, frightening, or moving. D Film historians do not believe that film reviews in newspapers and magazines reveal typical film audience members' views. E Films from the early part of the twentieth century were not usually reviewed in newspapers or magazines.

Correct answer: D. Film historians do not believe that film reviews in newspapers and magazines reveal typical film audience Question type: supported *** Rule: Answer D is correct because "film historians" often "find it difficult" to determine how people responded to films back then. If they believed that film reviews did reveal how people responded, then they wouldn't find it difficult to determine how people responded. Granted, this answer is not something that "must be true," but it's the "most strongly supported" answer. It's the answer that's most likely to be true.

In a recent study, one group of participants watched video recordings of themselves running on treadmills, and a second group watched recordings of other people running on treadmills. When contacted later, participants in the first group reported exercising, on average, 1 hour longer each day than did the other participants. This shows that watching a recording of yourself exercising can motivate you to exercise more. Which one of the following, if true, most weakens the argument? A In another study, people who watched recordings of themselves lifting weights exercised for more time each day than did people who watched recordings of themselves running. B Another study's members exhibited an increased willingness to give to charity after hearing stories in which people with whom they identified did so. C Participants who were already highly motivated to exercise did not report exercising for any longer each day than they had before the study. D In studies of identical twins, participants who observed their twin reading overreported by a significant amount how much time they themselves spent reading in the days that followed. E A third group of participants who watched recordings of themselves sitting on couches afterwards reported being sedentary for more time each day than did the other participants. Treadmills

Correct answer: D. In studies of identical twins, participants who observed their twin reading over reported by a significant amount how much time they themselves spent reading in the days that followed. Question type: Weaken ***** Rule Tricky one, but that doesn't mean we can't answer it correctly. The first thing to note about the study is that it uses self-reported data. Self-reported data can be correct, of course. But self-reported data can also be entirely bullshit. (Ask any doctor who asks their patients how many drinks they have in an average week.) This weakness turns out to be related to the correct answer. If you spot that weakness before going into the answer choices, you have a much better chance of avoiding the wrong answers and picking the right one. We're looking for a fact that, if true, would make the conclusion drawn from the study look worse. Look, I know many of you aren't going to be satisfied with this question. And I suppose that's okay. Give yourself permission to dislike maybe one question per two or three tests, that's probably fine. But I promise you that if you got 100 170+ scorers in the room, 90 or more of them would get this one right. You just can't pick any of the other answers. And the correct answer points out the weakness inherent in the interpretation of the study in the first place. The correct answer is D, and it's not bullshit. It's a totally valid question

Manager: The only employees who should receive bonuses this year are those who were exceptionally productive over the past year. Liang is an excellent account executive, but she works in a corporate division that has failed to meet its productivity goals for the year. Thus Liang should not receive a bonus this year. The reasoning in the manager's argument is flawed in that the argument A fails to take into account the possibility that the standards by which productivity is judged might vary across different divisions of a corporation B overlooks the possibility that a corporation as a whole can have a profitable year even though one division of the corporation does not C fails to justify its use of one group's performance as the basis for a conclusion about a wholly different group D reaches a conclusion about the performance of one member of a group merely on the basis of the performance of the group as a whole E takes for granted that an employee who has an unproductive year will not be exceptionally productive in subsequent years

Correct answer: D. Reaches a conclusion about the performance of one member of a group early on the basis of the performance of a group as a whole Question type- Flaw Rule find the apparent discrepancy in the argument. The argument is flawed if the evidence is not sufficient, and there are missing pieces, this is stuff that almost everyone agrees on.

Winston: The rules for awarding Nobel Prizes stipulate that no more than three people can share the same prize. Nobel Prizes in scientific disciplines are generally given in recognition of particular scientific results, however, and many important results are the work of four or more scientists. Sanjay: Those rules also stipulate that prize winners must be living, but some highly influential scientists died before their work was fully appreciated. The dialogue most strongly supports the claim that Winston and Sanjay agree that A the rules that govern the awarding of Nobel Prizes should be changed so that prizes can be awarded to deceased persons B the rules that govern the awarding of Nobel Prizes in scientific disciplines should be different from the rules for other Nobel Prizes C Nobel Prizes in scientific disciplines should not be given in recognition of particular scientific results D the evaluation of individual achievement in science is a highly subjective matter E Nobel Prizes are inaccurate indicators of scientists' contributions to their disciplines

Correct answer: E. Nobel Prizes are inaccurate indicators of scientists' contributions to their disciplines Question Type: Disagree***** Rule: One of these five answers is something that we have reason to believe the two authors agree upon. Which one? Yep. E would have been a clearer answer if it had said "not a full indicator of a scientist's contribution" instead of "inaccurate." But we can interpret "inaccurate" as "partially inaccurate," can't we? For example, let's say I'm going for the double 20 in darts but I hit the single 20 instead. That's both "accurate" (I hit the right number at least) and "inaccurate" (not quite where I wanted it) at the same time. "Inaccurate" doesn't have to be absolute, is what I'm saying. And we clearly can't pick A-D, so we can comfortably interpret E in that way.

Even though she thought the informant was untrustworthy, the journalist promised not to reveal his identity so long as the information he provided did not turn out to be false. However, she will publicly reveal the informant's identity if she is ordered to do so by a judge or her editor. After all, the information concerns safety violations at the power plant. Thus, the journalist will surely reveal the informant's identity even if the information is accurate. The conclusion of the argument follows logically if which one of the following is assumed? A The information that the informant provided is known to be false. B The journalist's editor will not order her to reveal the informant's identity unless the information is accurate and concerns public safety. C If the information concerns safety at the power plant, a judge will order the journalist to reveal her informant's identity. D The truth of the information provided by the informant can be verified only if the informant's identity is publicly revealed.

Correct answer: C. If the information concerns safety at the power plant, a judge will order the journalist to reveal her informants identity Question type- Sufficient Assumption Rule: Before you even get to the question stem, you should be trying to poke holes in the argument. In this case, the conclusion is that the journalist will reveal the informant's identity even if the information is accurate, but we never learn why she will. We are, however, given two reasons why the journalist could be forced to reveal the identity. Either because a judge or her editor told her to do so. Hence, before I even get to the question stem, I'm thinking, "this argument is flawed because we aren't actually given any reason why the journalist will reveal the information."

The mayor has been accused of taking a bribe based on the fact that a consultant that does business with the city paid for improvements to the mayor's vacation house. In his own defense, the mayor has said that he paid every bill for those improvements that was presented to him. Which one of the following, if true, most undermines the mayor's defense? A. Authorities are investigating the consultant for taking bribes from officials of other cities. B. The mayor was aware that many of the bills were being presented to the consultant rather than to the mayor. C. The building contractor in charge of the improvements to the mayor's house had done business with the city in the past. D. The improvements to the mayor's house were done with expensive materials and involved thousands of hours of labor. E. The amount of money that the city paid the consultant over the last year greatly exceeded the cost of the improvements to the mayor's house.

correct answer B. The mayor was aware that many of the bills were being presented to the consultant rather than to the mayor. question type- WEAKEN *** rule - (weaken the conclusion) First diesect premise and conclusion, Conclusion - The mayor has said that he paid every bill for those improvements that was presented to him. Main issue- It has to involve what was paid for and what wasn't. How to weaken the mayors argument.

Archaeologist: The earliest evidence of controlled fire use in Europe dates to just 400,000 years ago. This casts doubt on the commonly held view that, because of Europe's cold winter climate, mastery of fire was a necessary prerequisite for humans' migration there. Which one of the following is an assumption required by the argument? A The humans who first mastered fire used it for heat but not for cooking. B The climate in Europe was significantly colder 400,000 years ago than it is today. C Prior to 400,000 years ago, humans occasionally took advantage of naturally occurring fires. D Humans would not have mastered fire were it not for the need for heat in a cold climate. E There were humans inhabiting Europe prior to 400,000 years ago.

correct answer E. There were humans inhabiting Europe prior to 400,000 years ago. Question type- NECESSARY ASSUMPTION *** Rule- Premise author left out but absolutely needs. the wording is often weak. Main issue- 1. Because we don't have evidence it did not happen another assumption is that they migrated earlier then 400, 000 years ago. no one knows exactly when they migrated.


Conjuntos de estudio relacionados

CH 29: Management of Patients with Nonmalignant Hematologic Disorders

View Set

NURS 1446 Evolve Practice Questions - Chapter 1 MIDTERM

View Set

Professional Sales - MK 4330 Study Guide

View Set

FA and BM of the Ribs, sternum, and diaphragm

View Set

module 6 zybook 7.2 Arrays Array declarations and accessing elements

View Set